Practice Q's Exam #1 & #2 & #3

¡Supera tus tareas y exámenes ahora con Quizwiz!

b

A nurse assesses a client in the labor room and finds that the client's Bishop score for her cervical status is 6. Which medication may be administered to this client? a. Oxytocin b. Dinoprostone c. Mifepristone d. Methylergonovine

c

A nurse explains preterm labor to a group of nursing students. Which description of preterm labor indicates effective teaching? a. Preterm labor is defined as contractions during the delivery. b. Preterm labor is defined as contractions induced by prostaglandins. c. Preterm labor is defined as contractions between 20 and 36 weeks of gestation. d. Preterm labor is defined as contractions occurring before 20 weeks of gestation.

b

A nurse is caring for a client whose labor is being augmented with oxytocin. The nurse recognizes that the oxytocin should be discontinued immediately if there is evidence of: a. Uterine contractions occurring every 8 to 10 minutes. b. A fetal heart rate (FHR) of 180 with absence of variability. c. The client needing to void. d. Rupture of the client's amniotic membranes.

c

A nurse is caring for a postpartum client. Where does the nurse expect the fundus to be located if involution is progressing as expected 12 hours after birth? a. 2 cm below the umbilicus b. 3 cm above the umbilicus c. 1 cm above the umbilicus d. 3 cm below the umbilicus

c

A nurse is evaluating several obstetric patients for their risk for cervical insufficiency. Which patient would be considered to be most at risk? a. Primipara b. Grandmultip who has previously had all vaginal deliveries without a problem c. Primip who undergoes a cervical cone biopsy for cervical dysplasia prior to the pregnancy d. Multip who had her previous delivery via C section due to cephalopelvic disproportion (CPD)

c

A nurse is examining a patient who has been admitted for possible ectopic pregnancy who is approximately 8 weeks pregnant. Which finding would be a priority concern? a. No FHT heard via Doppler b. Scant vaginal bleeding noted on peri pad c. Ecchymosis noted around umbilicus d. Blood pressure 100/80

c

A nurse is monitoring a patient's reflexes (DTRs) while receiving magnesium sulfate therapy for treatment of preeclampsia. Which assessment finding indicates a cause for concern? a. Bilateral DTRs noted at 2+ b. DTRs response has been noted at 1+ since onset of therapy c. Positive clonus response elicited unilaterally d. Patient reports no pain upon examination of DTRs by nurse

d

A nurse is responsible for teaching new parents regarding the hygienic care of their newborn. Which instruction should the nurse provide regarding bathing? a. Avoid washing the head for at least 1 week to prevent heat loss. b. Sponge bathe the newborn for the first month of life. c. Cleanse the ears and nose with cotton-tipped swabs, such as Q-tips. d. Create a draft-free environment of at least 24° C (75° F) when bathing the infant.

a

A primiparous woman is to be discharged from the hospital the following day with her infant girl. Which behavior indicates a need for further intervention by the nurse before the woman can be discharged? a. The woman is disinterested in learning about infant care. b. The woman continues to hold and cuddle her infant after she has fed her. c. The woman reads a magazine while her infant sleeps. d. The woman changes her infant's diaper and then shows the nurse the contents of the diaper.

c

A woman presents to the emergency department complaining of bleeding and cramping. The initial nursing history is significant for a last menstrual period 6 weeks ago. On sterile speculum examination, the primary health care provider finds that the cervix is closed. The anticipated plan of care for this woman is based on a probable diagnosis of which type of spontaneous abortion? a. Incomplete b. Inevitable c. Threatened d. Septic

d

As part of the infant discharge instructions, the nurse is reviewing the use of the infant car safety seat. Which information is the highest priority for the nurse to share? a. Infant carriers are okay to use until an infant car safety seat can be purchased. b. For traveling on airplanes, buses, and trains, infant carriers are satisfactory. c. Infant car safety seats are used for infants only from birth to 15 pounds. d. Infant car seats should be rear facing and placed in the back seat of the car.

a,c,d

As recently as 2005, the AAP revised safe sleep practices to assist in the prevention of SIDS. The nurse should model these practices in the hospital and incorporate this information into the teaching plan for new parents. Which practices are ideal for role modeling? (Select all that apply.) a. Fully supine position for all sleep b. Side-sleeping position as an acceptable alternative c. "Tummy time" for play d. Infant sleep sacks or buntings e. Soft mattress

c

When assessing a neonate immediately after birth, the nurse observes an inability to close the eyes completely. The nurse also observes drooping of the corner of the neonate's mouth, and the absence of wrinkling of the forehead and nasolabial fold. What does the nurse infer from these findings? a. The neonate has bleeding in the subgaleal layer from labor. b. The neonate's cranial nerve V was pressurized during labor. c. The neonate's cranial nerve VII was pressurized during labor. d. The neonate was exposed to vaginal gonorrheal infection during labor.

c

When does a nurse caring for a client with eclampsia determine that the risk for another seizure has decreased? a. After birth occurs b. After labor begins c. 48 hours postpartum d. 24 hours postpartum

a

When should discharge instruction, or the teaching plan that tells the woman what she needs to know to care for herself and her newborn, officially begin? a. At the time of admission to the nurse's unit b. When the infant is presented to the mother at birth c. During the first visit with the physician in the unit d. When the take-home information packet is given to the couple

a

Which PPH conditions are considered medical emergencies that require immediate treatment? a. Inversion of the uterus and hypovolemic shock b. Hypotonic uterus and coagulopathies c. Subinvolution of the uterus and idiopathic thrombocytopenic purpura (ITP) d. Uterine atony and disseminated intravascular coagulation (DIC).

c

Which adaptation of the maternal-fetal exchange of oxygen occurs in response to uterine contraction? a. The maternal-fetal exchange of oxygen and waste products continues except when placental functions are reduced. b. This maternal-fetal exchange increases as the blood pressure decreases. c. It diminishes as the spiral arteries are compressed. d. This exchange of oxygen and waste products is not significantly affected by contractions.

d

Which basic type of pelvis includes the correct description and percentage of occurrence in women? a. Gynecoid: classic female pelvis; heart shaped; 75% b. Android: resembling the male pelvis; wide oval; 15% c. Anthropoid: resembling the pelvis of the ape; narrow; 10% d. Platypelloid: flattened, wide, and shallow pelvis; 3%

a

Which description of the four stages of labor is correct for both the definition and the duration? a. First stage: onset of regular uterine contractions to full dilation; less than 1 hour to 20 hours b. Second stage: full effacement to 4 to 5 cm; visible presenting part; 1 to 2 hours c. Third stage: active pushing to birth; 20 minutes (multiparous woman), 50 minutes (nulliparous woman) d. Fourth stage: delivery of the placenta to recovery; 30 minutes to 1 hour

a

Which explanation will assist the parents in their decision on whether they should circumcise their son? a. The circumcision procedure has pros and cons during the prenatal period. b. American Academy of Pediatrics (AAP) recommends that all male newborns be routinely circumcised. c. Circumcision is rarely painful, and any discomfort can be managed without medication. d. The infant will likely be alert and hungry shortly after the procedure.

a,b,d,e

Which factors influence cervical dilation? (Select all that apply.) a. Strong uterine contractions b. Force of the presenting fetal part against the cervix c. Size of the woman d. Pressure applied by the amniotic sac e. Scarring of the cervix

c,d,e

Which factors would lead to an increased likelihood of uterine rupture? Select all that apply. a. Preterm singleton pregnancy b. G3P3 with all vaginal deliveries c. Short interval between pregnancies d. Patient receiving a trial of labor (TOL) following a VBAC delivery e. Patient who had a primary caesarean section with a classic incision

c

Which indicator would lead the nurse to suspect that a postpartum patient experiencing hemorrhagic shock is getting worse? a. Restoration of blood pressure levels to normal range b. Capillary refill brisk c. Patient complaint of headache and increased reaction time to questioning d. Patient statement that she sees "stars"

b

Which information about variations in the infant's blood counts is important for the nurse to explain to the new parents? a. A somewhat lower-than-expected red blood cell count could be the result of a delay in clamping the umbilical cord. b. An early high white blood cell (WBC) count is normal at birth and should rapidly decrease. c. Platelet counts are higher in the newborn than in adults for the first few months. d. Even a modest vitamin K deficiency means a problem with the blood's ability to properly clot.

b,c,e

Which laboratory values would be found in a patient diagnosed with preeclampsia? (Select all that apply). a. Hemoglobin 8g/dL b. Platelet count of 75,000 c. LDH 100 units/L d. Burr cells e. BUN 25 mg/dL

d

Which nursing action promotes psychosocial development for a newborn? a. Washing hands prior to holding the newborn b. Measuring the newborn using an approved length board c. Weighing the newborn on the same scale during hospitalization d. Placing the newborn in the mother's arms during the first hour of life

b

Which of the following antihypertensive medications would cause a pregnant woman to have a positive Coombs test result? a. Nifedipine (Procardia) b. Methyldopa (Aldomet) c. Labetalol hydrochloride (Trandate) d. Hydralazine (Apresoline)

b

Which statement is the best rationale for assessing the maternal vital signs between uterine contractions? a. During a contraction, assessing the fetal heart rate is the priority. b. Maternal circulating blood volume temporarily increases during contractions. c. Maternal blood flow to the heart is reduced during contractions. d. Vital signs taken during contractions are not accurate.

a,b,c,e

Which statements describe the first stage of the neonatal transition period? (Select all that apply.) a. The neonatal transition period lasts no longer than 30 minutes. b. It is marked by spontaneous tremors, crying, and head movements. c. Passage of the meconium occurs during the neonatal transition period. d. This period may involve the infant suddenly and briefly sleeping. e. Audible grunting and nasal flaring may be present during this time.

a,b,c

Which statements regarding physiologic jaundice are accurate? (Select all that apply.) a. Neonatal jaundice is common; however, kernicterus is rare. b. Appearance of jaundice during the first 24 hours or beyond day 7 indicates a pathologic process. c. Because jaundice may not appear before discharge, parents need instruction on how to assess for jaundice and when to call for medical help. d. Jaundice is caused by reduced levels of serum bilirubin. e. Breastfed babies have a lower incidence of jaundice.

b, c

Which statements relate to preterm labor? Select all that apply. a. A premature baby has good cognitive development. b. The treatment for preterm labor includes bed rest and hydration. c. Preterm labor before the 20th week is indicative of a nonviable fetus. d. It is not desirable to stop the delivery in the case of preterm labor. e. Preterm labor refers to uterine contractions progressing to delivery before the 27th week of pregnancy.

b

With regard to preeclampsia and eclampsia, nurses should be aware that: a. Preeclampsia is a condition of the first trimester; eclampsia is a condition of the second and third trimesters. b. Preeclampsia results in decreased function in such organs as the placenta, kidneys, liver, and brain. c. The causes of preeclampsia and eclampsia are well documented. d. Severe preeclampsia is defined as preeclampsia plus proteinuria.

a,b

A nurse is teaching a class of expectant parents about changes that are to be expected during pregnancy. Which changes does the nurse explain result from the melanocyte-stimulating hormone? (Select all that apply) a. Chloasma b. Linea nigra c. Effacement d. Morning sickness e. Cervical softening f. Urinary frequency

d

A pregnant client with diabetes is referred to the dietitian in the prenatal clinic for nutritional assessment and counseling. What should the nurse emphasize when reinforcing the client's dietary program? a. The need to increase high-quality protein and decrease fats b. The need to increase carbohydrates to meet energy demands and prevent ketosis c. The need to eat a low-calorie diet that maintains the current insulin coverage and helps prevent hyperglycemia d. The need to eat a pregnancy diet that meets increased dietary needs and to adjust the insulin dosage as necessary

a

A woman with severe preeclampsia has been receiving magnesium sulfate by IV infusion for 8 hours. The nurse assesses the woman and documents the following findings: temperature 37.1° C, pulse rate 96 beats/min, respiratory rate 24 breaths/min, blood pressure 155/112 mm Hg, 3+ deep tendon reflexes, and no ankle clonus. The nurse calls the physician, anticipating an order for: a. hydralazine b. magnesium sulfate bolus c. diazepam d. calcium gluconate

a

The nurse in the birthing unit is caring for several postpartum clients. Which factor will increase the risk for hypotonic uterine dystocia? a. Twin gestation b. Gestational anemia c. Hypertonic contractions d. Gestational hypertension

b

When the fetal head begins to crown during an emergency precipitous birth, how should the nurse respond? a. Pressing firmly on the fundus b. Applying gentle perineal pressure c. Encouraging the client to push forcefully d. Telling the client to take prolonged deep breaths

a, c, d, e

Which statements are true regarding ectopic pregnancy? Select all that apply. a. Smoking is one of the risk factors for ectopic pregnancy. b. Ectopic pregnancy is directly related to fetopelvic incompatibility. c. Ectopic pregnancy occurs when the fertilized egg implants in the fallopian tubes. d. When a young women exhibits abdominal pain, ectopic pregnancy is suspected. e. If the adolescent exhibits abdominal pain and hypotension, the ectopic pregnancy may have ruptured.

a

Which tocolytic agent can be used safely without combining with other agents to inhibit labor and maintain a pregnancy? a. Nifedipine b. Terbutaline c. Indomethacin d. Magnesium sulfate

d

While caring for a woman who has had a positive contraction stress test (CST), what complication does the nurse suspect? a. Preeclampsia b. Placenta previa c. Imminent preterm birth d. Uteroplacental insufficiency

d

A client in active labor is admitted to the birthing room. A vaginal examination reveals that her cervix is dilated 6 to 7 cm. In light of this finding, what does the nurse expect? a. Client may experience nausea and vomiting. b. Client's bloody show will become more profuse. c. Client will experience uncontrollable shaking of her legs. d. Client's contractions will become longer and more frequent.

c

A pregnant woman with a history of heart disease visits the prenatal clinic toward the end of her second trimester. Which intervention does the nurse anticipate will be part of this client's care plan? a. Preparation for a cesarean birth b. Bed rest during the last trimester c. Prophylactic antibiotics at the time of birth d. Increasing dosages of cardiac medications as pregnancy progresses

a

A woman at 40 weeks' gestation is having contractions. Wondering whether she is in true labor, she asks, "How will you know if I'm really in labor?" Which information should the nurse provide to the patient at this time? a. The cervix dilates and becomes effaced in true labor. b. Bloody show is the first sign of true labor. c. The membranes rupture at the beginning of true labor. d. Fetal movements lessen and become weaker in true labor.

c

A woman's position is an important component of the labor progress. Which guidance is important for the nurse to provide to the laboring client? a. The supine position, which is commonly used in the United States, increases blood flow. b. The laboring client positioned on her hands and knees ("all fours" position) is hard on the woman's back. c. Frequent changes in position help relieve fatigue and increase the comfort of the laboring client. d. In a sitting or squatting position, abdominal muscles of the laboring client will have to work harder.

c

Which documentation on a woman's chart on postpartum day 14 indicates a normal involution process? a. Moderate bright red lochial flow b. Breasts firm and tender c. Fundus below the symphysis and nonpalpable d. Episiotomy slightly red and puffy

c

A client who is in labor is admitted 30 hours after her membranes ruptured. Which condition is this client at increased risk for? a. Cord prolapse b. Placenta previa c. Chorioamnionitis d. Abruptio placentae

c

The majority of ectopic pregnancies are located in the: a. Uterine fundus. b. Cervical os. c. Ampulla. d. Fimbriae.

c

A 36-year-old woman, G1 P0, is admitted to the labor and delivery unit for oxytocin induction. She is at 40 weeks' gestation. Which condition is a contraindication to the use of oxytocin induction? a. Chorioamnionitis b. Postterm pregnancy c. Active genital herpes infection d. Hypertension associated with pregnancy

d

A Gravida III, Para 0 is concerned about the potential outcome for this pregnancy because all of her prior pregnancies have resulted in stillborn deliveries. Which diagnostic test would help assess for fetal well-being now that her pregnancy is at 32 weeks gestation? a. Kleihauer-Betke test b. Chorionic villi sampling (CVS) c. Contraction stress test (CST) d. Ultrasound

c

A client admitted with preeclampsia is receiving magnesium sulfate. Which assessment finding indicates that a therapeutic level of the medication has been reached? a. Increased fetal activity b. Decreased urine output c. Deep tendon reflexes of +2 d. Respiratory rate of 10 breaths/min

a

A client at 24 weeks' gestation arrives at the clinic for a routine examination. She tells the nurse, "I feel puffy all over." In light of this statement, what is the nurse's most important assessment? a. Obtaining her blood pressure b. Determining how much salt she uses c. Asking the extent of her daily fluid intake d. Reviewing her history for total weight gain

a

A 1-day-old newborn has just expelled a thick, greenish-black stool. The nurse determines that this is the infant's first stool. What should the nurse do next? a. Document the stool in the infant's record. b. Send the stool to the laboratory per protocol. c. Assess the infant for an intestinal obstruction. d. Notify the health care provider that a tarry stool has been passed.

b

A 16-year-old primigravida at 32 weeks' gestation is admitted to the high-risk unit. Her blood pressure is 170/110 mm Hg and she has 4+ proteinuria. She has gained 50 lb (22.7 kg) during the pregnancy, and her face and extremities are edematous. Which complication is this client experiencing? a. Eclampsia b. Severe preeclampsia c. Chronic hypertension d. Gestational hypertension

a

A client gives birth to a full-term male with an 8/9 Apgar score. What should the immediate nursing care of this newborn include? a. Assessing respirations, keeping him warm, and identifying him b. Applying an antibiotic to the eyes, administering vitamin K, and bathing him c. Aspirating the oropharynx, rushing him to the nursery, and stimulating him often d. Weighing him, placing him in a crib, and waiting until the mother is ready to hold him

b

A client gives birth vaginally to an infant who weighs 8 lb, 13 oz (3997 g). An ice pack is applied to the perineum to ease the swelling and pain. The client complains, "This pain in my vagina and rectum is excruciating, and my vagina feels so full and heavy." What does the nurse suspect as the cause of the pain? a. Full bladder b. Vaginal hematoma c. Infected episiotomy d. Enlarged hemorrhoids

b

A client tells the nurse that the first day of her last menstrual period was July 22. What is the estimated date of birth (EDB)? a. May 7 b. April 29 c. April 22 d. March 6

b

A client who had tocolytic therapy for preterm labor is being discharged. Which instructions should the nurse include in the teaching plan? a. Restrict fluid intake b. Limit daily activities c. Monitor urine for protein d. Avoid deep-breathing exercises

a, c, e

A client with preeclampsia is admitted to the labor and birthing suite. Her blood pressure is 130/90 mm Hg, and she has 2+ protein in her urine along with edema of the hands and face. Which signs or symptoms would the client display if she were developing hemolysis, elevated liver enzymes, and low platelet count (HELLP syndrome)? Select all that apply. a. Headache b. Constipation c. Abdominal pain d. Vaginal bleeding e. Flulike symptoms

a

A client's labor has progressed to the point where she is 6 cm dilated; however, the fetal head is not engaged. An amniotomy is performed. After this procedure, the nurse checks the fetal heart rate. What other nursing action should be performed at this time? a. Inspecting the perineum b. Preparing for an immediate birth c. Measuring the maternal blood pressure d. Increasing the intravenous (IV) fluid rate

a

A client's membranes ruptured 20 hours before admission. The client was in labor for 24 hours before giving birth. For which postpartum complication is this client at risk? a. Infection b. Hemorrhage c. Uterine atony d. Amniotic fluid embolism

a

A mother is changing the diaper of her newborn son and notices that his scrotum appears large and swollen. The client is concerned. What is the best response from the nurse? a. "A large scrotum and swelling indicate a hydrocele, which is a common finding in male newborns." b. "I don't know, but I'm sure it is nothing." c. "Your baby might have testicular cancer." d. "Your baby's urine is backing up into his scrotum."

b

A patient tells the nurse about the funeral arrangements for her newborn son. The patient is thereby providing the nurse with information about: a. Grief process. b. Mourning process. c. Expression of loss. d. Family reaction.

d

A pregnant woman at 28 weeks of gestation has been diagnosed with gestational diabetes. The nurse caring for this client understands that: a. Oral hypoglycemic agents can be used if the woman is reluctant to give herself insulin. b. Dietary modifications and insulin are both required for adequate treatment. c. Glucose levels are monitored by testing urine four times a day and at bedtime. d. Dietary management involves distributing nutrient requirements over three meals and two or three snacks.

c

A pregnant woman has maternal phenylketonuria (PKU) and is interested in whether or not she will be able to breastfeed her baby. Which reaction by the nurse indicates accurate information? a. The patient can breastfeed the baby as long as she continues to maintain a PKU-restricted diet. b. The patient should alternate breastfeeding with bottle feeding in order to reduce PKU levels provided to the baby. c. The patient should be advised to not breastfeed the infant because her breast milk will contain large amounts of phenylalanine. d. The patient can breastfeed for the first 3 months without any untoward effects on the infant.

b

A woman with severe preeclampsia is being treated with an IV infusion of magnesium sulfate. This treatment is considered successful if: a. Blood pressure is reduced to prepregnant baseline. b. Seizures do not occur. c. Deep tendon reflexes become hypotonic. d. Diuresis reduces fluid retention.

d

Screening for critical congenital heart disease (CCHD) was added to the uniform screening panel in 2011. The nurse has explained this testing to the new mother. Which action by the nurse related to this test is correct? a. Screening is performed when the infant is 12 hours of age. b. Testing is performed with an electrocardiogram. c. Oxygen (O2) is measured in both hands and in the right foot. d. A passing result is an O2 saturation of 95%.

a,b,e

The "Period of Purple Crying" is a program developed to educate new parents about infant crying and the dangers of shaking a baby. Each letter in the acronym "PURPLE" represents a key concept of this program. Which concepts are accurate? (Select all that apply.) a. P: peak of crying and painful expression b. U: unexpected c. R: baby is resting at last d. L: extremely loud e. E: evening

b,c,d

The breast-feeding mother should be taught to expect which changes to the condition of the breasts? (Select all that apply.) a. Breast tenderness is likely to persist for approximately 1 week after the start of lactation. b. As lactation is established, a mass may form that can be distinguished from cancer by its positional shift from day to day. c. In nonlactating mothers, colostrum is present for the first few days after childbirth. d. If suckling is never begun or is discontinued, then lactation ceases within a few days to a week. e. Little change occurs to the breasts in the first 48 hours.

d

The nurse is caring for a client who is admitted to the birthing unit with a diagnosis of abruptio placentae. Which complication associated with a placental abruption should the nurse carefully monitor this client for? a. Cerebral hemorrhage b. Pulmonary edema c. Impending seizures d. Hypovolemic shock

c, e

What are the major complications noticed in adolescent girls during pregnancy? Select all that apply. a. Obesity b. Bleeding c. Ectopic pregnancy d. Gestational diabetes e. Spontaneous abortion

c

What is the rationale for the administration of vitamin K to the healthy full-term newborn? a. Most mothers have a diet deficient in vitamin K, which results in the infant being deficient. b. Vitamin K prevents the synthesis of prothrombin in the liver and must be administered by injection. c. Bacteria that synthesize vitamin K are not present in the newborn's intestinal tract. d. The supply of vitamin K in the healthy full-term newborn is inadequate for at least 3 to 4 months and must be supplemented.

a,b,e

fter an assessment of a male newborn, the nurse suspects postmaturity. Which observations help confirm this conclusion? (Select all that apply) a. Profuse scalp hair b. Parchmentlike skin c. Abundant vernix caseosa d. Few rugae over the scrotum e. Creases covering the entire soles

a,b,d,e

Which physiologic factors are reliable indicators of impending shock from postpartum hemorrhage? (Select all that apply.) a. Respirations b. Skin condition c. Blood pressure d. Level of consciousness e. Urinary output

c

A client in labor is admitted to the birthing unit. Assessment reveals that the fetus is in a footling breech presentation. What should the nurse consider regarding breech presentations when caring for this client? a. Severe back discomfort will occur. b. Length of labor usually is shortened. c. Cesarean birth probably will be necessary. d. Meconium in the amniotic fluid is a sign of fetal hypoxia.

d

A strict vegetarian (vegan) becomes pregnant and asks the nurse whether there is anything special she should do in regard to her diet during pregnancy. What is the most important measure for the nurse to instruct the client to take? a. Eat at least 40 g/day of protein. b. Drink at least 1 quart/day of milk. c. Take a vitamin supplemented with iron every day. d. Plan to eat from specific groups of vegetable proteins each day.

b

After an uneventful pregnancy a client at term arrives at the birthing unit. The nurse determines that her contractions are 10 minutes apart and that her cervix is dilated 2 cm. What stage of labor should the nurse document in the client's medical record? a. Second stage b. Latent first stage c. Active first stage d. Transition stage

d

How should the nurse interpret an Apgar score of 10 at 1 minute after birth? a. The infant is having no difficulty adjusting to extrauterine life and needs no further testing. b. The infant is in severe distress and needs resuscitation. c. The nurse predicts a future free of neurologic problems. d. The infant is having no difficulty adjusting to extrauterine life but should be assessed again at 5 minutes after birth.

a,d,e

Nurses play a critical role in educating parents regarding measures to prevent infant abduction. Which instructions contribute to infant safety and security? (Select all that apply.) a. The mother should check the photo identification (ID) of any person who comes to her room. b. The baby should be carried in the parent's arms from the room to the nursery. c. Because of infant security systems, the baby can be left unattended in the client's room. d. Parents should use caution when posting photographs of their infant on the Internet. e. The mom should request that a second staff member verify the identity of any questionable person.

d

Signs of a threatened abortion (miscarriage) are noted in a woman at 8 weeks of gestation. What is an appropriate management approach for this type of abortion? a. Prepare the woman for a dilation and curettage (D&C). b. Put the woman on bed rest for at least 1 week and reevaluate. c. Prepare the woman for an ultrasound and blood work. d. Comfort the woman by telling her that if she loses this baby, she may attempt to get pregnant again in 1 month.

a

The nurse admits a client in active labor to the birthing center. She is 100% effaced, dilated 3 cm, and at +1 station. What stage of labor has this client reached? a. First b. Latent c. Second d. Transitional

a,d,e

Which of the following presentations is associated with early pregnancy loss, occurring in less than 12 weeks gestation? (select all that apply). a. Chromosomal abnormalities b. Infection c. Cystitis d. Antiphospholipid syndrome e. Hypothyroidism f. Caffeine use

d

Upon arriving in the birthing room the nurse finds the client lying on her back with her head on a pillow and the bed in a flat position. The nurse explains that it is important to avoid lying in the supine position because of what reason? a. It may precipitate a severe headache. b. It can impede the progression of labor. c. It may cause nausea as labor progresses. d. It will prevent adequate blood flow to the fetus.

c

A woman comes into the clinic and states that she is thinking about becoming pregnant. What can the woman do to improve the health of her baby before she becomes pregnant? a. Go buy maternity clothes. b. Start running 3 miles (4.8 km) a day. c. Start taking prenatal vitamins. d. Buy a crib for the baby to sleep in.

b

The nurse is assessing several postpartum clients at the very beginning of her shift. Which problem does the nurse identify that might predispose a client to postpartum hemorrhage? a. Preeclampsia b. Multifetal pregnancy c. Prolonged first-stage labor d. Cephalopelvic disproportion

a

The nurse is assessing the Apgar scores of four different newborns in a pediatric ward. Which child does the nurse anticipate is experiencing severe distress? a. Newborn A b. Newborn B c. Newborn C d. Newborn D

d

Nursing assessment of a client in labor reveals that she is entering the transition phase of the first stage of labor. Which clinical manifestations support this conclusion? a. Facial redness and an urge to push b. Bulging perineum, crowning, and caput c. Less intense, less frequent contractions d. Increased bloody show, irritability, and shaking

a

Nursing follow-up care often includes home visits for the new mother and her infant. Which information related to home visits is correct? a. Ideally, the visit is scheduled within 72 hours after discharge. b. Home visits are available in all areas. c. Visits are completed within a 30-minute time frame. d. Blood draws are not a part of the home visit.

d

A woman gave birth vaginally to a 9-pound, 12-ounce girl yesterday. Her primary health care provider has written orders for perineal ice packs, use of a sitz bath three times daily, and a stool softener. Which information regarding the client's condition is most closely correlated with these orders? a. Woman is a gravida 2, para 2. b. Woman had a vacuum-assisted birth. c. Woman received epidural anesthesia. d. Woman has an episiotomy.

a

A first-time father is changing the diaper of his 1-day-old daughter. He asks the nurse, "What is this black, sticky stuff in her diaper?" What is the nurse's best response? a. "That's meconium, which is your baby's first stool. It's normal." b. "That's transitional stool." c. "That means your baby is bleeding internally." d. "Oh, don't worry about that. It's okay."

a

A hospital has a number of different perineal pads available for use. A nurse is observed soaking several of them and writing down what she sees. What goal is the nurse attempting to achieve by performing this practice? a. To improve the accuracy of blood loss estimation, which usually is a subjective assessment b. To determine which pad is best c. To demonstrate that other nurses usually underestimate blood loss d. To reveal to the nurse supervisor that one of them needs some time off

a

The nurse is caring for a client in her third trimester who is scheduled for an amniocentesis. What should the nurse do to prepare the client for this test? a. Instruct her to void immediately before the test. b. Tell her to assume the high Fowler position before the test. c. Encourage her to drink three glasses of water before the test. d. Advise her to take nothing by mouth for several hours before the test.

a

The nurse is preparing to administer a hepatitis B virus (HBV) vaccine to a newborn. Which intervention by the nurse is correct? a. Obtaining a syringe with a 25-gauge, 5/8-inch needle for medication administration b. Confirming that the newborn's mother has been infected with the HBV c. Assessing the dorsogluteal muscle as the preferred site for injection d. Confirming that the newborn is at least 24 hours old

a

A pregnant woman's amniotic membranes rupture. Prolapsed cord is suspected. Which intervention is the nurse's top priority? a. Place the woman in the knee-chest position. b. Cover the cord in a sterile towel saturated with warm normal saline. c. Prepare the woman for a cesarean birth. d. Start oxygen by face mask.

d

A primigravida at 12 weeks' gestation complains of nausea and vomiting during a visit to the prenatal clinic. Which pregnancy hormone should the nurse explain is thought to be responsible for nausea and vomiting during the first trimester? a. Estrogen b. Progesterone c. Human placental lactogen (hPL) d. Human chorionic gonadotropin (hCG)

b

A woman with severe preeclampsia is receiving a magnesium sulfate infusion. The nurse becomes concerned after assessment when the woman exhibits: a. A sleepy, sedated affect. b. A respiratory rate of 10 breaths/min. c. Deep tendon reflexes of 2+. d. Absence of ankle clonus.

c

During an emergency birth the fetal head is crowning on the perineum. How should the nurse support the head as it is being delivered? a. By applying suprapubic pressure b. By placing a hand firmly against the perineum c. By distributing the fingers evenly around the head d. By maintaining pressure against the anterior fontanel

b

In which clinical situation would cervical ripening drugs be prescribed to pregnant women? a. There is a need for a termination of pregnancy. b. The cervical status indicates a Bishop score of 6. c. Uterine contractions occur after 25 weeks of gestation. d. Uterine contractions occur after 18 weeks of gestation.

d

The nurse is caring for a postpartum client who has experienced an abruptio placentae. Which assessment indicates that disseminated intravascular coagulation (DIC) is occurring? a. Boggy uterus b. Hypovolemic shock c. Multiple vaginal clots d. Bleeding at the venipuncture site

d

A 25-year-old multiparous woman gave birth to an infant boy 1 day ago. Today her husband brings a large container of brown seaweed soup to the hospital. When the nurse enters the room, the husband asks for help with warming the soup so that his wife can eat it. What is the nurse'smost appropriate response? a. "Didn't you like your lunch?" b. "Does your physician know that you are planning to eat that?" c. "What is that anyway?" d. "I'll warm the soup in the microwave for you."

c

A client at 7 weeks' gestation elects to undergo an induced abortion. After receiving oral mifepristone, she returns to the clinic 2 days later to have misoprostol inserted vaginally. How long after the procedure should the nurse have the patient return for a follow-up visit? a. 4 hours b. 8 to 24 hours c. 4 to 8 days d. 2 weeks

c

A first-time dad is concerned that his 3-day-old daughter's skin looks "yellow." In the nurse's explanation of physiologic jaundice, what fact should be included? a. Physiologic jaundice occurs during the first 24 hours of life. b. Physiologic jaundice is caused by blood incompatibilities between the mother and the infant blood types. c. Physiologic jaundice becomes visible when serum bilirubin levels peak between the second and fourth days of life. d. Physiologic jaundice is also known as breast milk jaundice.

a

A new mother is concerned that her 1-month-old infant is nursing every 2 hours. Which response by the nurse is most appropriate? a. "It's common for newborns to nurse this often. Let's talk about how you're adjusting with the new baby." b. "Breast milk is easily digested; giving your infant a little rice cereal will keep him full longer." c. "It sounds as though your baby is a little spoiled; try to resist feeding more often than every 4 hours." d. "You may not be producing enough milk; it'll be important for you to supplement feedings with formula."

c

A nurse is assessing a newborn girl who is 2 hours old. Which finding warrants a call to the health care provider? a. Blood glucose of 45 mg/dl using a Dextrostix screening method b. Heart rate of 160 beats per minute after vigorously crying c. Laceration of the cheek d. Passage of a dark black-green substance from the rectum

d

A nurse is assessing a newborn whose mother had a precipitate birth at home. For which complication should the nurse assess the newborn? a. Facial palsy b. Dislocated hip c. Fractured clavicle d. Intracranial hemorrhage

c

A patient who is pregnant already has Type 2 diabetes with a hemoglobin A1c value of 7. The nurse would categorize this patient as having: a. Gestational diabetes. b. Insulin-dependent diabetes complicated by pregnancy. c. Pregestational diabetes mellitus. d. Non-insulin-dependent diabetes with complications.

d

A pregnant woman was prescribed a drug for pregnancy-induced hypertension. Later, the client developed muscle weakness, edema, and nausea for which calcium gluconate was administered. What drug was administered to the client to treat pregnancy-induced hypertension? a. Nifedipine b. Terbutaline c. Indomethacin d. Magnesium sulfate

c

A pregnant woman who is at 21 weeks of gestation has an elevated blood pressure of 140/98. Past medical history reveals that the woman has been treated for hypertension. On the basis of this information, the nurse would classify this patient as having: a. preeclampsia b. gestational hypertension c. superimposed preeclampsia d. chronic hypertension

b

A pregnant woman with cardiac disease is informed about signs of cardiac decompensation. She should be told that the earliest sign of decompensation is most often: a. orthopnea b. decreasing energy levels c. moist frequent cough and frothy sputum d. crackles (rales) at the bases of the lungs on auscultation

d

A woman at 39 weeks of gestation with a history of preeclampsia is admitted to the labor and birth unit. She suddenly experiences increased contraction frequency to every 1 to 2 minutes; dark red vaginal bleeding; and a tense, painful abdomen. The nurse suspects the onset of: a. Eclamptic seizure. b. Rupture of the uterus. c. Placenta previa. d. Placental abruption.

a

A woman diagnosed with marginal placenta previa gave birth vaginally 15 minutes ago. At present she is at the greatest risk for: a. Hemorrhage. b. Infection. c. Urinary retention. d. Thrombophlebitis.

c

Complicated bereavement: a. Occurs when, in multiple births, one child dies and the other or others live. b. Is a state in which the parents are ambivalent, as with an abortion. c. Is an extremely intense grief reaction that persists for a long time. d. Is felt by the family of adolescent mothers who lose their babies.

b

Diabetes in pregnancy puts the fetus at risk in several ways. Nurses should be aware that: a. With good control of maternal glucose levels, sudden and unexplained stillbirth is no longer a major concern. b. The most important cause of perinatal loss in diabetic pregnancy is congenital malformations. c. Infants of mothers with diabetes have the same risks for respiratory distress syndrome because of the careful monitoring. d. At birth, the neonate of a diabetic mother is no longer in any greater risk.

c

During a physical examination in the prenatal clinic the client's vaginal mucosa is noted to have a purplish discoloration. Which sign should the nurse document in the client's clinical record? a. Hegar b. Goodell c. Chadwick d. Braxton Hicks

a,b,c,e

During life in utero, oxygenation of the fetus occurs through transplacental gas exchange. When birth occurs, four factors combine to stimulate the respiratory center in the medulla. The initiation of respiration then follows. What are these four essential factors? (Select all that apply) a. Chemical b. Mechanical c. Thermal d. Psychologic e. Sensory

b

Five minutes after a birth the nurse determines that the client's placenta is separating. Which clinical finding indicates placental separation? a. Uterine fundus relaxes b. Umbilical cord lengthens c. Abdominal pain becomes severe d. Vaginal seepage of blood is continuous

d

For a woman at 42 weeks of gestation, which finding requires more assessment by the nurse? a. Fetal heart rate of 116 beats/min b. Cervix dilated 2 cm and 50% effaced c. Score of 8 on the biophysical profile d. One fetal movement noted in 1 hour of assessment by the mother

a,c,d,e

Hearing loss is one of the genetic disorders included in the universal screening program. Auditory screening of all newborns within the first month of life is recommended by the AAP. What is the rationale for having this testing performed? (Select all that apply.) a. Prevents or reduces developmental delays b. Reassures concerned new parents c. Provides early identification and treatment d. Helps the child communicate better e. Is recommended by the Joint Committee on Infant Hearing

b

How should nurses provide effective nursing care to clients from different cultural backgrounds? a. By advising clients that some cultural practices may be harmful to health b. By providing care that fits the clients' cultural beliefs c. By strictly adhering to organization policies regarding nursing care d. By ignoring the cultural aspect and focusing on the medical aspect of care

a

How would the nurse optimally reassure the parents of an infant who develops a cephalhematoma? a. A cephalhematoma may occur with a spontaneous vaginal birth. b. A cephalhematoma only happens as a result of a forceps- or vacuum-assisted delivery. c. It is present immediately after birth. d. The blood will gradually absorb over the first few months of life.

a

In caring for the woman with disseminated intravascular coagulation (DIC), what order should the nurse anticipate? a. Administration of blood products. b. Preparation of the woman for invasive hemodynamic monitoring c. Restriction of intravascular fluids d. Administration of steroids

b,c,d

In which situations would the use of Methergine or prostaglandin be contraindicated even if the patient was experiencing a postpartum significant bleed? (Select all that apply). a. Patient has delivered twin pregnancies. b. Patient's blood pressure postpartum is 180/90. c. Patient has a history of asthma. d. Patient has a mitral valve prolapse. e. Patient is a grand multip.

c

Nurses should be aware that HELLP syndrome: a. Is a mild form of preeclampsia. b. Can be diagnosed by a nurse alert to its symptoms. c. Is characterized by hemolysis, elevated liver enzymes, and low platelets. d. Is associated with preterm labor but not perinatal mortality.

d

Parents who have not already done so need to make time for newborn follow-up of the discharge. According to the American Academy of Pediatrics (AAP), when should a breastfeeding infant first need to be seen for a follow-up examination? a. 2 weeks of age b. 7 to 10 days after childbirth c. 4 to 5 days after hospital discharge d. 48 to 72 hours after hospital discharge

b,d,e

Postpartum fatigue (PPF) is more than just feeling tired. It is a complex phenomenon affected by physiologic, psychologic, and situational variables. Which factors contribute to this phenomenon? (Select all that apply.) a. Precipitous labor b. Hospital routines c. Bottle feeding d. Anemia e. Excitement

c

The first and most important nursing intervention when a nurse observes profuse postpartum bleeding is to: a. Call the woman's primary health care provider. b. Administer the standing order for an oxytocic. c. Palpate the uterus and massage it if it is boggy. d. Assess maternal blood pressure and pulse for signs of hypovolemic shock.

a

The laboratory results for a postpartum woman are as follows: blood type, A; Rh status, positive; rubella titer, 1:8 (enzyme immunoassay [EIA] 0.8); hematocrit, 30%. How should the nurse best interpret these data? a. Rubella vaccine should be administered. b. Blood transfusion is necessary. c. Rh immune globulin is necessary within 72 hours of childbirth. d. Kleihauer-Betke test should be performed.

b

The priority assessment in evaluating a pregnant woman with severe nausea and vomiting is: a. Fasting blood glucose level. b. Ketonuria. c. Bilirubin. d. White blood cell count.

d

The trend in the United States is for women to remain hospitalized no longer than 1 or 2 days after giving birth. Which scenario is not a contributor to this model of care? a. Wellness orientation model of care rather than a sick-care model b. Desire to reduce health care costs c. Consumer demand for fewer medical interventions and more family-focused experiences d. Less need for nursing time as a result of more medical and technologic advances and devices available at home that can provide information

b

Thirty minutes after a client gives birth, the nurse palpates the client's uterus. It is relaxed and the lochia is excessive. What is the nurse's initial action? a. Check vital signs. b. Massage the uterus. c. Notify the practitioner. d. Elevate the foot of the bed.

a, c

Typical signs of neonatal abstinence syndrome related to opioid withdrawal usually begin within 24 hours after birth. What characteristics should the nurse anticipate in the infant of a suspected or known drug abuser? Select all that apply. a. Tremors b. Dehydration c. Hyperactivity d. Muscle hypotonicity e. Prolonged sleep periods

c

Under the Newborns' and Mothers' Health Protection Act, all health plans are required to allow new mothers and newborns to remain in the hospital for a minimum of _____ hours after a normal vaginal birth and for _____ hours after a cesarean birth. What is the correct interpretation of this legislation? a. 24; 72 b. 24; 96 c. 48; 96 d. 48; 120

c, d, e

What drugs are used to induce abortion in an adolescent? Select all that apply. a. Leuprolide b. Zidovudine c. Misoprostol d. Mifepristone e. Methotrexate

c

Which drug may be used both for cervical ripening during labor and as a stomach protectant? a. Raloxifene b. Clomiphene c. Misoprostol d. Dinoprostone

c

After the birth of her child, a mother tells the nurse, "I was told that my baby has to have an injection of vitamin K. Why is this necessary?" How should the nurse respond? a. "Your baby needs the injection to promote development of red blood cells." b. "An injection of vitamin K will help keep your baby from becoming jaundiced." c. "Newborns are deficient in vitamin K. This treatment will ensure adequate levels." d. "A newborn's blood clots extremely rapidly. This injection will help decrease the clotting time."

d

As part of their teaching function at discharge, nurses should educate parents regarding safe sleep. Based on the most recent evidence, which information is incorrect and should be discussed with parents? a. Prevent exposure to people with upper respiratory tract infections. b. Keep the infant away from secondhand smoke. c. Avoid loose bedding, water beds, and beanbag chairs. d. Place the infant on his or her abdomen to sleep.

b

At 1 minute after birth a nurse assesses an infant and notes a heart rate of 80 beats per minute, some flexion of extremities, a weak cry, grimacing, and a pink body but blue extremities. Which Apgar score does the nurse calculate based upon these observations and signs? a. 4 b. 5 c. 6 d. 7

a

At 37 weeks' gestation a client's membranes spontaneously rupture; however, she does not have any labor contractions. What action is most important in the nursing plan of care for this client? a. Monitoring for the presence of fever b. Monitoring for signs of preeclampsia c. Monitoring for heavy vaginal bleeding d. Making preparations for fetal scalp pH sampling

a,b,d

Because of its size and rigidity, the fetal head has a major effect on the birth process. Which bones comprise the structure of the fetal skull? (Select all that apply.) a. Parietal b. Temporal c. Fontanel d. Occipital e. Femoral

a

While a client is being interviewed on her first prenatal visit she states that she has a 4-year-old son who was born at 41 weeks' gestation and a 3-year-old daughter who was born at 35 weeks' gestation. The client lost one pregnancy at 9 weeks and another at 18 weeks. Using the GTPAL system, how would you record this information? a. G5 T1 P1 A2 L2 b. G4 T1 P1 A2 L2 c. G4 T2 P0 A0 L2 d. G5 T2 P1 A1 L2

c

While assessing the integument of a 24-hour-old newborn, the nurse notes a pink papular rash with vesicles superimposed on the thorax, back, and abdomen. What action is the highest priority for the nurse to take at this time? a. Immediately notify the physician. b. Move the newborn to an isolation nursery. c. Document the finding as erythema toxicum neonatorum. d. Take the newborn's temperature, and obtain a culture of one of the vesicles.

d

While evaluating the reflexes of a newborn, the nurse notes that with a loud noise the newborn symmetrically abducts and extends his arms, his fingers fan out and form a C with the thumb and forefinger, and he has a slight tremor. The nurse would document this finding as a positive _____ reflex. a. tonic neck b. glabellar (Myerson) c. Babinski d. Moro

b

The most prevalent clinical manifestation of abruptio placentae (as opposed to placenta previa) is: a. Bleeding. b. Intense abdominal pain. c. Uterine activity. d. Cramping.

d

Certain changes stimulate chemoreceptors in the aorta and carotid bodies to prepare the fetus for initiating respirations immediately after birth. Which change in fetal physiologic activity isnot part of this process? a. Fetal lung fluid is cleared from the air passages during labor and vaginal birth. b. Fetal partial pressure of oxygen (PO2) decreases. c. Fetal partial pressure of carbon dioxide in arterial blood (PaCO2) increases. d. Fetal respiratory movements increase during labor.

a

What should be included in the nursing care for a client at 41 weeks' gestation who is scheduled for a contraction stress test? a. Having the client empty her bladder b. Placing the client in a supine position c. Informing the client about the need for cesarean birth d. Preparing the client for insertion of an internal monitor

a, c, e

A client is admitted to the birthing suite with a blood pressure of 150/90 mm Hg, 3+ proteinuria, and edema of the hands and face. A diagnosis of severe preeclampsia is made. What other clinical findings support this diagnosis? Select all that apply. a. Headache b. Constipation c. Abdominal pain d. Vaginal bleeding e. Visual disturbances

c

A client has been receiving oxytocin to augment labor. For what adverse reaction caused by a prolonged oxytocin infusion should the nurse monitor the client? a. Change in affect b. Hyperventilation c. Water intoxication d. Increased temperature

b

A primiparous woman is watching her newborn sleep. She wants him to wake up and respond to her. The mother asks the nurse how much he will sleep every day. What is an appropriate response by the nurse? a. "He will only wake up to be fed, and you should not bother him between feedings." b. "The newborn sleeps approximately 17 hours a day, with periods of wakefulness gradually increasing." c. "He will probably follow your same sleep and wake patterns, and you can expect him to be awake soon." d. "He is being stubborn by not waking up when you want him to. You should try to keep him awake during the daytime so that he will sleep through the night."

c

A recently delivered mother and her baby are at the clinic for a 6-week postpartum checkup. Which response by the client alerts the nurse that psychosocial outcomes have not been met? a. The woman excessively discusses her labor and birth experience. b. The woman feels that her baby is more attractive and clever than any others. c. The woman has not given the baby a name. d. The woman has a partner or family members who react very positively about the baby.

a

A woman gave birth to a 7-pound, 6-ounce infant girl 1 hour ago. The birth was vaginal and the estimated blood loss (EBL) was 1500 ml. When evaluating the woman's vital signs, which finding would be of greatest concern to the nurse? a. Temperature 37.9° C, heart rate 120 beats per minute (bpm), respirations 20 breaths per minute, and blood pressure 90/50 mm Hg b. Temperature 37.4° C, heart rate 88 bpm, respirations 36 breaths per minute, and blood pressure 126/68 mm Hg c. Temperature 38° C, heart rate 80 bpm, respirations 16 breaths per minute, and blood pressure 110/80 mm Hg d. Temperature 36.8° C, heart rate 60 bpm, respirations 18 breaths per minute, and blood pressure 140/90 mm Hg

c

A woman has been administered prostaglandin E 2 as a part of a treatment to induce labor. Which factor determines if an infusion with oxytocin should be continued? a. An unripe cervix b. One contraction per minute c. One contraction every three minutes d. Cervix dilation increasing 2 cm/hour

a,d,e

An infant born at 36 weeks' gestation weighs 4 lb, 3 oz (1899 g) and has Apgar scores of 7 and 9. Which nursing actions will be performed upon the infant's admission to the nursery? (Select all that apply) a. Recording of vital signs b. Administration of oxygen c. Offering a bottle of dextrose in water d. Evaluation of the neonate's health status e. Supportive measures to keep the neonate's body temperature stable

c

An infant is admitted to the nursery after a difficult shoulder dystocia vaginal birth. Which condition should the nurse carefully assess this newborn for? a. Facial paralysis b. Cephalhematoma c. Brachial plexus injury d. Spinal cord syndrome

c,d

Changes in blood volume after childbirth depend on several factors such as blood loss during childbirth and the amount of extravascular water (physiologic edema) mobilized and excreted. What amount of blood loss does the postpartum nurse anticipate? (Select all that apply.) a. 100 ml b. 250 ml or less c. 300 to 500 ml d. 500 to 1000 ml e. 1500 ml or greater

a

During a routine prenatal office visit at 26 weeks' gestation, a client states that she is getting fat all over and that she even needed to buy bigger shoes. What is the next nursing action? a. Obtaining the client's weight and blood pressure b. Reassuring the client that weight gain is expected c. Supporting the client's decision to buy comfortable shoes d. Teaching the client about the importance of limiting fatty foods and sweets

a

During her sixth month of pregnancy, a woman visits the prenatal clinic for the first time. As part of the initial assessment a complete blood count and urinalysis are performed. Which laboratory finding should alert the nurse to the need for further assessment? a. Hemoglobin of 10 g/dL (100 mmol/L) b. Urine specific gravity of 1.020 c. Glucose level of 1+ in the urine d. White blood cell count of 9000/mm 3 (9 × 10 9/L)

a,b,d

If a woman is at risk for thrombus and is not ready to ambulate, which nursing intervention would the nurse use? (Select all that apply.) a. Putting her in antiembolic stockings (thromboembolic deterrent [TED] hose) and/or sequential compression device (SCD) boots b. Having her flex, extend, and rotate her feet, ankles, and legs c. Having her sit in a chair d. Immediately notifying the physician if a positive Homans sign occurs e. Promoting bed rest

a

In many hospitals, new mothers are routinely presented with gift bags containing samples of infant formula. This practice is inconsistent with what? a. Baby Friendly Hospital Initiative b. Promotion of longer periods of breastfeeding c. Perception of being supportive to both bottle feeding and breastfeeding mothers d. Association with earlier cessation of breastfeeding

d

In planning for the care of a 30-year-old woman with pregestational diabetes, the nurse recognizes that the most important factor affecting pregnancy outcome is the: a. Mother's age. b. Number of years since diabetes was diagnosed. c. Amount of insulin required prenatally. d. Degree of glycemic control during pregnancy.

a

The nurse is caring for four clients on the postpartum unit. Which client will most likely state that she is having difficulty sleeping due to afterbirth pains? a. Multipara who has vaginally delivered three children b. Primipara whose newborn weighed 7 lb c. Multipara with effectively controlled diabetes d. Multipara whose second child was small for gestational age

d

On her first visit to the prenatal clinic a woman is to have a pelvic examination. What information should the nurse include when discussing the examination? a. She should direct her questions to the healthcare provider. b. She should relax during the examination to prevent discomfort. c. A douche will be necessary before the examination for the biopsy. d. A rectal examination may be performed after the pelvic examination.

a,b,c,d

Pain should be regularly assessed in all newborns. If the infant is displaying physiologic or behavioral cues that indicate pain, then measures should be taken to manage the pain. Which interventions are examples of nonpharmacologic pain management techniques? (Select all that apply.) a. Swaddling b. Nonnutritive sucking c. Skin-to-skin contact with the mother d. Sucrose e. Acetaminophen

b

The nurse is completing a physical examination of the newborn 24 hours after birth. Which component of the evaluation is correct? a. The parents are excused to reduce their normal anxiety. b. The nurse can gauge the neonate's maturity level by assessing his or her general appearance. c. Once often neglected, blood pressure is now routinely checked. d. When the nurse listens to the neonate's heart, the S1 and S2 sounds can be heard; the S1sound is somewhat higher in pitch and sharper than the S2 sound.

a

The nurse is providing care to a multiparous client in active labor. The client is requesting something for the pain. What is the nurse's priority intervention? a. Examining the client's cervix for dilation and effacement b. Determining the client's options by assessing the prescriptions in the chart c. Asking her whether she prefers an epidural or something in her intravenous line d. Evaluating the fetal monitoring strip to determine the frequency and duration of contractions

a

The nurse caring for a newborn checks the record to note clinical findings that occurred before her shift. Which finding related to the renal system would be of increased significance and require further action? a. The pediatrician should be notified if the newborn has not voided in 24 hours. b. Breastfed infants will likely void more often during the first days after birth. c. Brick dust or blood on a diaper is always cause to notify the physician. d. Weight loss from fluid loss and other normal factors should be made up in 4 to 7 days.

b

The nurse has received a report regarding a client in labor. The woman's last vaginal examination was recorded as 3 cm, 30%, and -2. What is the nurse's interpretation of this assessment? a. Cervix is effaced 3 cm and dilated 30%; the presenting part is 2 cm above the ischial spines. b. Cervix is dilated 3 cm and effaced 30%; the presenting part is 2 cm above the ischial spines. c. Cervix is effaced 3 cm and dilated 30%; the presenting part is 2 cm below the ischial spines. d. Cervix is dilated 3 cm and effaced 30%; the presenting part is 2 cm below the ischial spines.

b

The nurse is providing discharge teaching to the parents of a 3-day-old infant. The mother expresses concern regarding sudden infant death syndrome (SIDS). To reduce the risk of SIDS during sleep, how does the nurse instruct the parents to position the infant? a. Prone b. Supine c. Side-lying d. Next to an adult in bed for closer monitoring

a

The nurse is using the New Ballard Scale to determine the gestational age of a newborn. Which assessment finding is consistent with a gestational age of 40 weeks? a. Flexed posture b. Abundant lanugo c. Smooth, pink skin with visible veins d. Faint red marks on the soles of the feet

d

Which finding indicates that a newborn has vernix caseosa? a. Brown hair on the skin b. Rosy to yellowish skin c. Light-pink to reddish-brown skin d. Cheese-like substance on the skin

a

A nurse caring for pregnant women must be aware that the most common medical complication of pregnancy is: a. Hypertension. b. Hyperemesis gravidarum. c. Hemorrhagic complications. d. Infections.

d

Which infant response to cool environmental conditions is either not effective or not available to them? a. Constriction of peripheral blood vessels b. Metabolism of brown fat c. Increased respiratory rates d. Unflexing from the normal position

b

Nurses can help their clients by keeping them informed about the distinctive stages of labor. Which description of the phases of the first stage of labor is accurate? a. Latent: Mild, regular contractions; no dilation; bloody show; duration of 2 to 4 hours b. Active: Moderate, regular contractions; 4- to 7-cm dilation; duration of 3 to 6 hours c. Lull: No contractions; dilation stable; duration of 20 to 60 minutes d. Transition: Very strong but irregular contractions; 8- to 10-cm dilation; duration of 1 to 2 hours

d

The nurse is caring for a high-risk pregnant client who has had a positive contraction stress test (CST). What would the nurse interpret the result to mean? a. A nonstress test is needed. b. An immediate cesarean birth is needed. c. The fetal heart is within the expected limits for the average fetus. d. Late decelerations of the fetal heart rate are occurring with each contraction.

a

The nurse is caring for a patient who has just had an amniotomy performed by the primary healthcare provider. The fetal heart rate immediately decreases from 140 to 80 beats/min. What is the priority nursing action? a. Inspecting the vagina b. Administering oxygen c. Increasing the intravenous fluids d. Placing the client in the knee-chest position

c

What should the nurse's next action be if the client's white blood cell (WBC) count is 25,000/mm3 on her second postpartum day? a. Immediately inform the physician. b. Have the laboratory draw blood for reanalysis. c. Recognize that this count is an acceptable range at this point postpartum. d. Immediately begin antibiotic therapy.

a

The nurse is assessing a 12-hour-old newborn. Which clinical finding should be reported to the health care provider in a timely manner? a. Jaundice b. Cephalhematoma c. Erythema toxicum d. Edematous genitalia

a,b,e

The nurse is assessing a male newborn. Which characteristics should alert the nurse to conclude that the newborn is a preterm infant? (Select all that apply) a. Small breast buds b. Wrinkled thin skin c. Multiple sole creases d. Presence of scrotal rugae e. Pinnae that remain flat when folded

a

While assessing a client during the fourth stage of labor a nurse notes that the perineal pad is soaked end to end with approximately 75 mL of lochia rubra. What is the priority nursing action? a. Palpating the uterine fundus b. Documenting the amount and type of lochia c. Accompanying the client to the bathroom to empty her bladder d. Calling the laboratory to test the hemoglobin and hematocrit levels

a

While caring for a client during labor, what does the nurse remember about the second stage of labor? a. It ends at the time of birth. b. It ends as the placenta is expelled. = c. It begins with the transition phase of labor. d. It begins with the onset of strong contractions.

a

While caring for a pregnant client with a body mass index of 32 during labor, the nurse observes that the second stage of labor lasts for about 11 minutes. The nurse also finds that the expected birth weight of the fetus is around 4200 g. Which complication does the nurse anticipate in the neonate after birth? a. Erb palsy b. Klumpke palsy c. Strawberry hemangioma d. Erythema toxicum neonatorum

c

A nurse is assessing a client at 16 weeks' gestation. Where does the nurse expect the fundal height to be located? a. Above the umbilicus b. At the level of the umbilicus c. Half the distance to the umbilicus d. Slightly above the symphysis pubis

a

Which newborn reflex is elicited by stroking the lateral sole of the infant's foot from the heel to the ball of the foot? a. Babinski b. Tonic neck c. Stepping d. Plantar grasp

a

A 3.8-kg infant was vaginally delivered at 39 weeks of gestation after a 30-minute second stage. A nuchal cord occurred. After the birth, the infant is noted to have petechiae over the face and upper back. Based on the nurse's knowledge, which information regarding petechiae should be shared with the parents? a. Petechiae (pinpoint hemorrhagic areas) are benign if they disappear within 48 hours of childbirth. b. These hemorrhagic areas may result from increased blood volume. c. Petechiae should always be further investigated. d. Petechiae usually occur with a forceps delivery.

c

A 37-year-old woman agrees to have a prenatal test done in order to diagnose fetal defects. There is a history of Down syndrome in her family. Which invasive prenatal test provides the earliest diagnosis and rapid test results? a. Nonstress test b. Amniocentesis c. Chorionic villus sampling d. Percutaneous umbilical blood sampling

b,c,e

A nurse is working with a diabetic patient who recently found out she is pregnant. In coordinating an interdisciplinary team to help manage the patient throughout the pregnancy, the nurse would include: (select all that apply). a. Family practice physician b. Dietician c. Perinatologist d. Occupational therapist e. Nephrologist f. Speech therapist

a

A woman in labor hears the primary healthcare provider tell the nurse that the fetal lie is longitudinal. The mother asks the nurse what this means in relation to her labor and birth of the baby. How should the nurse respond? a. "A vaginal birth is possible." b. "We're anticipating a cesarean delivery." c. "It has no relevance to the labor and birth." d. "Labor probably will be long, and you might have back pain."

a

After assessing a neonate immediately after birth who was delivered using forceps, the nurse confirms facial paralysis. Which information does the nurse provide to the mother? a. "Don't panic; it will resolve within a few days." b. "Take the newborn to a neurologist immediately." c. "The infant requires phototherapy for a few minutes." d. "Refrain from breast-feeding the infant for a few days."

b

After the removal of a hydatidiform mole, the nurse assesses the client's laboratory data during a follow-up visit. The nurse notes that a prolonged increase of the serum human chorionic gonadotropin (hCG) level is a danger sign. Which condition is this client at increased risk of developing? a. Uterine rupture b. Choriocarcinoma c. Hyperemesis gravidarum d. Disseminated intravascular coagulation (DIC)

b

An infant is born precipitously in the emergency department. What should the nurse's initial action be? a. Tie and cut the umbilical cord b. Establish an airway for the newborn c. Ascertain the condition of the uterine fundus d. Arrange transport for mother and infant to the birthing unit

d

At 39 weeks' gestation a client asks the nurse about the difference between true and false labor. Which information regarding true labor contractions should the nurse include in a response to the client's question? a. Usually fluctuate in length b. Continuous, without relaxation c. Related to time of membrane rupture d. Accompanied by progressive cervical dilation

d

At a client's first prenatal visit, the nurse-midwife performs a pelvic examination. The nurse states that the client's cervix is bluish purple, which is known as the Chadwick sign. The client becomes concerned and asks whether something is wrong. What does the nurse respond with about this expected finding? a. "It helps confirm your pregnancy." b. "It is not unusual, even in women who are not pregnant." c. "It occurs because the blood is trapped by the pregnant uterus." d. "It is caused by increased blood flow to the uterus during pregnancy."

b

The nurse is assessing a pregnant client at the end of her second trimester. Which clinical finding causes the nurse to suspect that the client has preeclampsia? a. Progressive weight gain b. Two urine samples showing proteinuria c. Dependent ankle edema during the late afternoon d. Blood pressure fluctuations on three successive measurements

d

The nurse is obtaining the health history of a woman who is visiting the prenatal clinic for the first time. She states that she is 5 months pregnant. Which positive sign of pregnancy should the nurse evaluate in this client? a. Quickening b. Enlarged abdomen c. Cervical color change d. Audible fetal heartbeat

b

The parents of a newborn ask the nurse how much the newborn can see. The parents specifically want to know what type of visual stimuli they should provide for their newborn. What information provided by the nurse would be most useful to these new parents? a. "Infants can see very little until approximately 3 months of age." b. "Infants can track their parents' eyes and can distinguish patterns; they prefer complex patterns." c. "The infant's eyes must be protected. Infants enjoy looking at brightly colored stripes." d. "It's important to shield the newborn's eyes. Overhead lights help them see better."

b

The parents of a newborn with phenylketonuria (PKU) ask a nurse how to prevent future problems. What must the nurse consider before responding? a. Most important is diagnosis within 2 days after birth b. Most important is the institution of a corrective formula soon after birth c. It depends on whether phenylpyruvic acid is found in the urine 1 week after birth d. It depends on the level of phenylalanine found in the blood immediately after birth

b

What information should the nurse understand fully regarding rubella and Rh status? a. Breastfeeding mothers cannot be vaccinated with the live attenuated rubella virus. b. Women should be warned that the rubella vaccination is teratogenic and that they must avoid pregnancy for at least 1 month after vaccination. c. Rh immunoglobulin is safely administered intravenously because it cannot harm a nursing infant. d. Rh immunoglobulin boosts the immune system and thereby enhances the effectiveness of vaccinations.

d

Which statement is most likely to be associated with a breech presentation? a. Least common malpresentation b. Descent rapid c. Diagnosis by ultrasound only d. High rate of neuromuscular disorders

d

A client in the high-risk postpartum unit has had a precipitous labor and birth. Which maternal complication should the nurse anticipate? a. Hypertension b. Hypoglycemia c. Chilling and shivering d. Bleeding and infection

b

A client is admitted for induction of labor. An intravenous infusion of oxytocin is started. When the client's contractions begin they are 1.5 to 2 minutes in duration. While the nurse is in the room, one contraction lasts 3 minutes. What should the nurse do first? a. Give oxygen by nasal cannula. b. Turn off the oxytocin infusion. c. Reposition the monitoring belts. d. Place a call light next to the client.

c

A multipara whose membranes have ruptured is admitted in early labor. Assessment reveals a breech presentation, cervical dilation of 3 cm, and fetal station at -2. For what complication should the nurse assess when caring for this client? a. Vaginal bleeding b. Urinary tract infection c. Prolapse of the umbilical cord d. Meconium in the amniotic fluid

d

A client at 32 weeks' gestation is admitted to the prenatal unit in preterm labor. An infusion of magnesium sulfate is started. What physiologic response indicates to the nurse that the magnesium sulfate is having a therapeutic effect? a. Dilation of the cervix by 1 cm every hour b. Tightening and pain in the perineal area c. A decrease in blood pressure to 120/80 mm Hg d. A decrease in frequency and duration of contractions

a,b,e

A client at 43 weeks' gestation has just given birth to an infant with typical postmaturity characteristics. Which signs of postmaturity does the nurse identify? (Select all that apply) a. Cracked and peeling skin b. Long scalp hair and fingernails c. Red, puffy appearance of face and neck d. Vernix caseosa covering the back and buttocks e. Creases covering the neonate's full soles and palms

d

A client in active labor arrives in the birthing unit, and birth is imminent. What is the most important question for the nurse to ask at this time? a. "Is this your first baby?" b. "Have your membranes ruptured?" c. "When did your contractions begin?" d. "When is your baby's expected date of birth?"

a,b,c

A client in her 10th week of pregnancy exhibits presumptive signs of pregnancy. Which clinical findings may the nurse determine upon assessment? (Select all that apply) a. Amenorrhea b. Breast changes c. Urinary frequency d. Abdominal enlargement e. Positive urine pregnancy test

d

A client in labor at 39 weeks' gestation is told by the primary healthcare provider that she will require a cesarean delivery. The nurse reviews the client's prenatal history. What preexisting condition is the most likely reason for the cesarean birth? a. Gonorrhea b. Chlamydia c. Chronic hepatitis d. Active genital herpes

c

A client in labor is admitted to the birthing unit 20 hours after her membranes have ruptured. Which complication should the nurse anticipate when assessing the character of the client's amniotic fluid? a. Cord prolapse b. Placenta previa c. Maternal sepsis d. Abruptio placentae

b

A client in labor is admitted with a suspected breech presentation. Which occurrence should the nurse be prepared for? a. Uterine inertia b. Prolapsed cord c. Imminent birth d. Precipitate labor

c

A client in the active phase of the first stage of labor begins to tremble, becomes very tense during contractions, and is quite irritable. She frequently states, "I can't take this a minute longer." What does this behavior indicate to the nurse caring for her? a. There was no preparation for labor. b. She should receive an analgesic for pain. c. She is entering the transition phase of labor. d. Hypertonic uterine contractions are developing.

a

A client is warm and asks for a fan in her room for her comfort. The nurse enters the room to assess the mother and her infant and finds the infant unwrapped in his crib with the fan blowing over him on high. The nurse instructs the mother that the fan should not be directed toward the newborn and that the newborn should be wrapped in a blanket. The mother asks why. How would the nurse respond? a. "Your baby may lose heat by convection, which means that he will lose heat from his body to the cooler ambient air. You should keep him wrapped, and should prevent cool air from blowing on him." b. "Your baby may lose heat by conduction, which means that he will lose heat from his body to the cooler ambient air. You should keep him wrapped, and should prevent cool air from blowing on him." c. "Your baby may lose heat by evaporation, which means that he will lose heat from his body to the cooler ambient air. You should keep him wrapped, and should prevent cool air from blowing on him." d. "Your baby will easily get cold stressed and needs to be bundled up at all times."

c

A client who has had a cesarean birth appears upset. She has been having difficulty breastfeeding for two days and now asks the nurse to bring her a bottle of formula. What is the nurse's initial action? a. Obtaining the requested formula b. Administering the prescribed pain medication c. Assessing the client's breastfeeding technique d. Notifying the practitioner of the client's request to switch feeding methods

c

A client who is 38 weeks pregnant presents to the labor unit for a nonstress test (NST). The resulting fetal monitor strip is shown. How does the nurse interpret this finding? a. Negative because of the lack of contractions b. Nonreassuring; fetal heart rate lacks variability c. Reassuring; fetal heart rate accelerates with movement d. Positive; demonstrates decelerations with fetal movement

c, d

A client who is at risk for seizures as a result of severe preeclampsia is receiving an intravenous infusion of magnesium sulfate. What findings cause the nurse to determine that the client is showing signs of magnesium sulfate toxicity? Select all that apply. a. Proteinuria b. Epigastric pain c. Respirations of 10 breaths/min d. Loss of patellar reflexes e. Urine output of 40 mL/hr

c

A client whose membranes have ruptured is admitted to the birthing unit. Her cervix is dilated 3 cm and 50% effaced. The amniotic fluid is clear, and the fetal heart rate is stable. Which outcome does the nurse anticipate? a. A prolonged second stage of labor b. A difficult birth resulting from delayed effacement c. Birth of the fetus within a day d. The stimulation of labor with an oxytocin infusion

c

A client with hypertension has labor pains before the 35th week of gestation. Which pharmacologic intervention does the nurse anticipate to be beneficial for this client? a. Preparing the client for an abortion b. Administer terbutaline c. Administer magnesium sulfate d. Administer sedatives and maintain hydration

a

A client's membranes rupture while her labor is being augmented with an oxytocin infusion. The nurse observes variable decelerations in the fetal heart rate on the fetal monitor strip. Which action should the nurse initiate next? a. Changing the client's position b. Taking the client's blood pressure c. Stopping the client's oxytocin infusion d. Preparing the client for an immediate birth

c

A man calls the prenatal clinic to ask the nurse when he should bring his wife to the hospital. He says, "The baby is due in 2 weeks, but she thinks it could be earlier. This is our first baby, and we're nervous." The nurse knows that as a nullipara, it would be important for the client to be seen if the contractions do what? a. Decrease when the client walks b. Are irregular and vary in intensity c. Come every 5 minutes for an hour d. Come every 10 minutes for an hour

c

A mother expresses fear about changing her infant's diaper after he is circumcised. What does the client need to be taught to care for her newborn son? a. Cleanse the penis with prepackaged diaper wipes every 3 to 4 hours. b. Apply constant, firm pressure by squeezing the penis with the fingers for at least 5 minutes if bleeding occurs. c. Gently cleanse the penis with water and apply petroleum jelly around the glans after each diaper change. d. Wash off the yellow exudate that forms on the glans at least once every day to prevent infection.

b

A new father wants to know what medication was put into his infant's eyes and why it is needed. How does the nurse explain the purpose of the erythromycin (Ilotycin) ophthalmic ointment? a. Erythromycin (Ilotycin) ophthalmic ointment destroys an infectious exudate caused byStaphylococcus that could make the infant blind. b. This ophthalmic ointment prevents gonorrheal and chlamydial infection of the infant's eyes, potentially acquired from the birth canal. c. Erythromycin (Ilotycin) prevents potentially harmful exudate from invading the tear ducts of the infant's eyes, leading to dry eyes. d. This ointment prevents the infant's eyelids from sticking together and helps the infant see.

a

A newborn's birth was prolonged because the fetal shoulders were very wide. Which reflex does the nurse anticipate a problem with? a. Moro b. Plantar c. Babinski d. Stepping

a

A nonstress test (NST) is scheduled for a client with mild preeclampsia. During the test, the client asks the nurse what it means when the fetal heart rate goes up every time the fetus moves. How should the nurse respond? a. "These accelerations are a sign of fetal well-being." b. "These accelerations indicate fetal head compression." c. "Umbilical cord compression is causing these accelerations." d. "Uteroplacental insufficiency is causing these accelerations."

d

A nursing student is helping the nursery nurses with morning vital signs. A baby born 10 hours ago by cesarean section is found to have moist lung sounds. What is the best interpretation of these data? a. The nurse should immediately notify the pediatrician for this emergency situation. b. The neonate must have aspirated surfactant. c. If this baby was born vaginally, then a pneumothorax could be indicated. d. The lungs of a baby delivered by cesarean section may sound moist during the first 24 hours after childbirth.

d

A postpartum client who was receiving an intravenous infusion of oxytocin to stimulate labor asks the nurse why it is not being discontinued now that the baby is born. How should the nurse respond? a. "The oxytocin promotes the flow of lochia." b. "The oxytocin eases the discomfort of involution." c. "The oxytocin enhances the healing of tissue in the uterus." d. "The contractions prevent excessive bleeding."

a

A pregnant client arrives at the prenatal clinic, and the nurse obtains her obstetrical history. The client has two children at home, one born at 38 weeks' gestation and the second born at 34 weeks' gestation. She has also had one miscarriage, at 18 weeks, and an elective abortion. Using the GTPAL system, what is the client's obstetrical record? a. G5 T1 P1 A2 L2 b. G4 T2 P2 A1 L4 c. G2 T3 P3 A2 L1 d. G3 T2 P1 A3 L3

a

A pregnant client arrives on the birthing unit from the emergency department with frank blood running down both legs and a reported low blood pressure. What is the priority nursing intervention? a. Assessing fetal heart tones b. Assessing for a prolapsed cord c. Starting an intravenous (IV) infusion d. Inserting a uterine pressure catheter

d

A pregnant client in the first trimester is experiencing nausea and vomiting. What does the nurse determine about this discomfort? a. It is always present during early pregnancy. b. It will disappear when lightening occurs. c. It is a common response to an unwanted pregnancy. d. It may be related to an increased human chorionic gonadotropin level.

a

A pregnant client in the third trimester tells the nurse in the prenatal clinic that she is experiencing heartburn after every meal. Which explanation should the nurse provide regarding the cause of the heartburn? a. "The esophageal sphincter relaxes and allows acid to be regurgitated." b. "In pregnancy, gastric motility increases, causing a burning sensation." c. "In pregnancy, gastric pH increases, causing acid to enter the esophagus." d. "In pregnancy, the pyloric sphincter relaxes, allowing acid to enter the intestine."

b

A pregnant client tells the nurse that her husband is a chain smoker. What information should the nurse's teaching include? a. Secondhand smoke is related to an increased blood level of alpha-fetoprotein. b. Continued exposure to secondhand smoke is related to fetal growth restriction. c. If the mother does not smoke, the fetus will not be affected by secondhand smoke. d. If the mother is not in the same room where there is smoking, the fetus is not affected.

b,c,e

A pregnant patient experiences thyroid storm following delivery of her infant. What interventions would the nurse anticipate to be ordered by the physician? (select all that apply). a. Restriction of intravenous fluids to prevent fluid overload b. Administration of oxygen c. Antipyretics d. Synthroid e. PTU

c

A pregnant woman at 14 weeks of gestation is admitted to the hospital with a diagnosis of hyperemesis gravidarum. The primary goal of her treatment at this time is to: a. Rest the gastrointestinal (GI) tract by restricting all oral intake for 48 hours. b. Reduce emotional distress by encouraging the woman to discuss her feelings. c. Reverse fluid, electrolyte, and acid-base imbalances. d. Restore the woman's ability to take and retain oral fluid and foods.

c

At a routine monthly visit, while assessing a client who is in her 26th week of gestation, the nurse identifies the presence of striae gravidarum. The nurse describes this condition to the client as what? a. Brownish blotches on the face b. Purplish discoloration of the cervix c. Reddish streaks on the abdomen and breasts d. A black line running between the umbilicus and mons veneris

d

What is the priority nursing intervention during the first 2 hours after a cesarean birth? a. Evaluating fluid needs to maintain optimum hydration b. Monitoring the incision to help prevent the onset of infection c. Encouraging bonding to promote mother-infant interaction d. Assessing the lochia to identify the complication of hemorrhage

b

Which of the following findings is not likely to be seen in a pregnant patient who has hypothyroidism? a. miscarriage b. macrosomia c. gestational hypertension d. placental abruption

b

The nurse is caring for a primigravid client during labor. Which physiologic finding does the nurse observe that indicates birth is about to take place? a. Bloody discharge from the vagina is increasing. b. The perineum has begun to bulge with each contraction. c. The client becomes irritable and stops following instructions. d. Contractions occur more frequently, are stronger, and last longer.

c

The nurse is teaching new parents about metabolic screening for the newborn. Which statement is most helpful to these clients? a. All states test for phenylketonuria (PKU), hypothyroidism, cystic fibrosis, and sickle cell diseases. b. Federal law prohibits newborn genetic testing without parental consent. c. If genetic screening is performed before the infant is 24 hours old, then it should be repeated at age 1 to 2 weeks. d. Hearing screening is now mandated by federal law.

c

The nurse should be cognizant of which important information regarding the gastrointestinal (GI) system of the newborn? a. The newborn's cheeks are full because of normal fluid retention. b. The nipple of the bottle or breast must be placed well inside the baby's mouth because teeth have been developing in utero, and one or more may even be through. c. Regurgitation during the first day or two can be reduced by burping the infant and slightly elevating the baby's head. d. Bacteria are already present in the infant's GI tract at birth because they traveled through the placenta.

a

The nurse should be cognizant of which important statement regarding care of the umbilical cord? a. The stump can become easily infected. b. If bleeding occurs from the vessels of the cord, then the nurse should immediately call for assistance. c. The cord clamp is removed at cord separation. d. The average cord separation time is 5 to 7 days.

d

The partner of a woman in labor is having difficulty timing the frequency of contractions and asks the nurse to review the procedure. How should contractions be timed? a. From the end of one contraction to the end of the next contraction b. From the end of one contraction to the beginning of the next contraction c. From the beginning of one contraction to the end of the next contraction d. From the beginning of one contraction to the beginning of the next contraction

a,c,d

Which practices contribute to the prevention of postpartum infection? (Select all that apply.) a. Not allowing the mother to walk barefoot at the hospital b. Educating the client to wipe from back to front after voiding c. Having staff members with conditions such as strep throat, conjunctivitis, and diarrhea stay home d. Instructing the mother to change her perineal pad from front to back each time she voids or defecates e. Not permitting visitors with cough or colds to enter the postpartum unit

a

Which presentation is accurately described in terms of both the resenting part and the frequency of occurrence? a. Cephalic: occiput, at least 96% b. Breech: sacrum, 10% to 15% c. Shoulder: scapula, 10% to 15% d. Cephalic: cranial, 80% to 85%

a

Which reflex does the nurse assess in a newborn to determine auditory ability? a. Startle reflex b. Rooting reflex c. Glabellar reflex d. Extrusion reflex

c

Which statement accurately describes an appropriate-for-gestational age (AGA) weight assessment? a. AGA weight assessment falls between the 25th and 75th percentiles for the infant's age. b. AGA weight assessment depends on the infant's length and the size of the newborn's head. c. AGA weight assessment falls between the 10th and 90th percentiles for the infant's age. d. AGA weight assessment is modified to consider intrauterine growth restriction (IUGR).

b

Which statement by a newly delivered woman indicates that she knows what to expect regarding her menstrual activity after childbirth? a. "My first menstrual cycle will be lighter than normal and then will get heavier every month thereafter." b. "My first menstrual cycle will be heavier than normal and will return to my prepregnant volume within three or four cycles." c. "I will not have a menstrual cycle for 6 months after childbirth." d. "My first menstrual cycle will be heavier than normal and then will be light for several months after."

c

Which statement by the client would lead the nurse to believe that labor has been established? a. "I passed some thick, pink mucus when I urinated this morning." b. "My bag of waters just broke." c. "The contractions in my uterus are getting stronger and closer together." d. "My baby dropped, and I have to urinate more frequently now."

b

A small-for-gestational-age (SGA) newborn who has just been admitted to the nursery has a high-pitched cry, appears jittery, and exhibits irregular respirations. What complication does the nurse suspect? a. Hypovolemia b. Hypoglycemia c. Hypercalcemia d. Hypothyroidism

a,b,e

A woman is being seen in the prenatal clinic at 36 weeks' gestation. The nurse is reviewing signs and symptoms that should be reported to the primary healthcare provider with the mother. Which signs and symptoms require further evaluation by the primary healthcare provider? (Select all that apply) a. Decreased urine output b. Blurred vision with spots c. Urinary frequency without dysuria d. Heartburn after eating a fatty meal e. Contractions that are regular and 5 minutes apart f. Shortness of breath after climbing a flight of stairs

d

During labor an internal fetal monitor is applied. Which fetal heart rate (FHR) should most concern the nurse? a. One that does not slow during contractions b. One that ranges from 130 to 140 beats/min c. One that drops to 110 beats/min during a contraction d. One that returns to baseline after a contraction ends

a

How does the nurse distinguish true labor from false labor? a. Cervical dilation is evident. b. Contractions stop when the client walks around. c. The client's contractions progress only when she is in a side-lying position. d. Contractions occur immediately after the membranes rupture.

d

A client is scheduled for a nonstress test in the 37th week of gestation. The nurse explains the procedure. Which statement demonstrates that the client understands the teaching? a. "I'll need to have an IV so the medication can be injected before the test." b. "My baby may get very restless after I have this test." c. "I hope this test doesn't cause my labor to start too early." d. "If the heart reacts well, my baby should do OK when I give birth."

d

A client's membranes rupture spontaneously during the latent phase of the first stage of labor, and the fluid is greenish brown. What does the nurse conclude? a. Infection is present b. Cesarean birth is necessary c. Precipitate birth is imminent d. The fetus may be compromised in utero

d

After a spontaneous vaginal delivery the client expresses concern when the newborn is brought to her after being cleaned and examined, regarding a red rash with small papules on the face, chest, and back. What condition does the nurse recognize? a. Harlequin sign b. Vernix caseosa c. Nevus flammeus d. Erythema toxicum

d

The nurse is caring for a client in labor. Which assessment finding reveals that the transition phase of labor has probably begun? a. The client assumes the lithotomy position. b. The frequency of contractions decreases. c. The client complains of back and perineal pain. d. The client begins to perspire and has a flushed face.

d

A pregnant client tells the nurse that she thinks she has developed an allergy because her nose is often very congested and she has difficulty breathing. How should the nurse reply? a. "Use a nasal decongestant at least twice a day." b. "It is common for allergies to develop during pregnancy." c. "That is not normal; you may have a chronic respiratory infection." d. "That is an expected occurrence; the increased hormones are responsible for the congestion."

b

A pregnant client who is scheduled for a nonstress test (NST) asks a nurse how the test can show that "my baby is all right." The nurse explains that it is a way of evaluating the condition of the fetus by comparing the fetal heart rate (FHR) with what? a. Fetal gestational age b. Fetal physical activity c. Maternal blood pressure d. Maternal uterine contractions

d

A pregnant client's blood test reveals an increased alpha-fetoprotein (AFP) level. Which condition does the nurse suspect that this result indicates? a. Cystic fibrosis b. Phenylketonuria c. Down syndrome d. Neural tube defect

b

A pregnant woman tells a nurse, "I think I can feel the baby move now. It feels like butterflies in my stomach. My friend calls it feeling life." What term should the nurse include when discussing fetal movement with the woman? a. Lightening b. Quickening c. Engagement d. Ballottement

c

A primigravida at 8 weeks' gestation is visiting the prenatal clinic for the first time. Which finding would an assessment reveal at this time? a. Lightening b. Quickening c. Goodell sign d. Braxton Hicks sign

a,c,d

A registered nurse is teaching a nursing student about structural factors that affect labor in adolescents. Which statements by the nursing student indicate effective learning? (Select all that apply) a. "Fetopelvic incompatibility results from the teenager's smaller stature." b. "Twelve-year-old girls generally have labors that are shorter than older women's." c. "Cephalopelvic disproportion is the main reason for cesarean births in adolescents." d. "Prolonged labor in an adolescent aged 13 years is the result of fetopelvic incompatibility." e. "The transition from pelvic disproportion to pelvic adequacy usually occurs around age 18."

d

A mother asks the neonatal nurse why her infant must be monitored so closely for hypoglycemia when her type 1 diabetes was in excellent control during the entire pregnancy. How should the nurse best respond? a. "A newborn's glucose level drops after birth, so we're being especially cautious with your baby because of your diabetes." b. "A newborn's pancreas produces an increased amount of insulin during the first day of birth, so we're checking to see whether hypoglycemia has occurred." c. "Babies of mothers with diabetes do not have large stores of glucose at birth, so it's difficult for them to maintain the blood glucose level within an acceptable range." d. "Babies of mothers with diabetes have a higher-than-average insulin level because of the excess glucose received from the mothers during pregnancy, so the glucose level may drop."

d

A multigravida client has a spontaneous vaginal birth. Five minutes later the placenta is expelled. Where does a nurse expect to locate the uterine fundus at this time? a. In the pelvic cavity b. Just below the xiphoid process c. At the umbilicus and in the right quadrant d. Halfway between the symphysis pubis and the umbilicus

d

A nurse is obtaining a health history from a primigravida on her first visit to the prenatal clinic. Before discussing the client's health habits with her, what does the nurse consider the most important factor in the survival of the client's newborn? a. Reproductive history b. Adequacy of prenatal care c. Health habits and social class d. Gestational age and birth weight

c

During the initial assessment of a dark-skinned neonate the nurse observes several dark round areas on a newborn's buttocks. How should this observation be documented? a. Stork bites b. Forceps marks c. Mongolian spots d. Ecchymotic areas

a

Prolonged labor is an incident very common in pregnant adolescents. Which age group of adolescents has the highest risk of prolonged labor? a. 12 to 13 years b. 14 to 15 years c. 16 to 18 years d. 19 to 21 years

b

The nurse explains the purpose of a nonstress test to a pregnant client who is at 39 weeks' gestation. This test is a way of evaluating the condition of the fetus by comparing the fetal heart rate with what? a. Fetal lie b. Fetal movement c. Maternal blood pressure d. Maternal uterine contractions

b

The nurse is assessing a new mother at a healthcare facility. Which symptom does the nurse identify as a risk factor for postpartum blues? a. Frantic energy b. Mild irritability c. Hallucinations d. Unwillingness to sleep

a

The nurse is assessing a newborn immediately after birth. Which finding indicates normal development? a. A body weight of 3500 g b. Blood pressure of 70/60 mm Hg c. A core body temperature of 96° F (35.6° C) d. Head circumference 3 cm less than chest circumference

b,c,e

The nurse is assessing the newborn in the first hour after birth. Which findings does the nurse identify as normal for the newborn? (Select all that apply) a. The newborn has a flat abdomen. b. The newborn weighs 6 lbs (2,700 g). c. The newborn's hands and feet appear cyanosed. d. The newborn does not blink in the presence of light. e. The circumference of the head is 33 cm (13 in).

c

The primary healthcare provider determines that a fetus is in a breech presentation. Which complication should the nurse monitor this client for? a. Rapid dilation of the cervix, indicating precipitate labor b. Stronger contractions, indicating progression of the labor c. Nonreassuring fetal signs, indicating prolapse of the cord d. Cessation of contractions, indicating primary uterine inertia

b,c,e

What complications are associated with excessive weight gain during pregnancy in adolescents? (Select all that apply). a. Fetal anemia b. Preterm labor c. Cesarean delivery d. Maternal mortality e. Postpartum obesity

c

When palpating a client's fundus on the second postpartum day, the nurse determines that it is above the umbilicus and displaced to the right. What does the nurse conclude? a. There is a slow rate of involution. b. There are retained placental fragments. c. The bladder has become overdistended. d. The uterine ligaments are overstretched.

a

When performing a postoperative assessment, which parameter would alert the nurse to a common side effect of epidural anesthesia? a. Decreased blood pressure b. Increased oral temperature c. Diminished peripheral pulses d. Unequal bilateral breath sounds

a

Which clinical finding does the nurse anticipate regarding the alveoli in the lungs of a 28-week-gestation neonate? a. They have a tendency to collapse with each breath. b. There usually is a sufficient supply of pulmonary surfactant. c. Although apparently mature they cannot absorb adequate oxygen. d. Oxygen is not released into the circulation because they overinflate.

a,c,d,e

Which of the following variables are scored on a biophysical profile? (Select all that apply) a. Fetal tone b. Fetal position c. Fetal movement d. Amniotic fluid index e. Fetal breathing movements f. Contraction stress test results

c

While caring for a client who gave birth 1 day ago, the nurse determines that the client's uterine fundus is firm at one fingerbreadth below the umbilicus, blood pressure is 110/70 mm Hg, pulse is 72 beats per minute, and respirations are 16 breaths per minute. The client's perineal pad is saturated with lochia rubra. What is the priority nursing action? a. Recording these expected findings b. Obtaining an order for an oxytocic medication c. Asking the client when she last changed the perineal pad d. Notifying the primary healthcare provider that the client may be hemorrhaging


Conjuntos de estudio relacionados

Glycogen Metabolism & the Pentose Phosphate Pathway 9.3

View Set

Smartbook: Chapter 7 Accounting Information Systems

View Set

Health Psych Chapter 4 Adherence

View Set

Chapter 08-QUIZ-Consideration of Internal Control in an Information Technology Environment

View Set

Differentiate factors that determine readiness to learn from those that determine ability to learn.

View Set

Lesson 5 English Synoyms, Antonyms, and Analogies

View Set